XV математическая олимпиада «Шелковый путь», 2016 год


Пусть $a$, $b$ и $c$ такие действительные числа, что $\left| \left( a-b \right)\left( b-c \right)\left( c-a \right) \right|=1.$ Найдите наименьшее значение выражения $\left| a \right|+\left| b \right|+\left| c \right|$. ( Сатылханов К. )
посмотреть в олимпиаде

Комментарий/решение:

пред. Правка 2   1
2016-04-27 20:50:17.0 #

Решение одного из участников:

Пусть $x,y,z=|a|,|b|,|c|$ соответственно. Тогда по неравенству Шура:

$x^3+y^3+z^3+6xyz \ge x^3+y^3+z^3+3xyz \ge xy(x+y)+yz(y+z)+zx(z+x)$

Прибавив к обеем сторонам

$3(xy(x+y)+yz(y+z)+zx(z+x))$

получим

$$(x+y+z)^3\ge 4(xy(x+y)+yz(y+z)+zx(z+x))\ge $$

$$4|a^2b+b^2c+c^2a-ab^2-bc^2-ca^2|=4|(a-b)(b-c)(c-a)|=4.$$

Откуда

$(|a|+|b|+|c|)^3\ge 4$

Остается лишь найти пример.

  3 | проверено модератором
2016-04-30 23:58:31.0 #

Т.к $GM\leq AM$,то

$(a-b)(b-c)\leq (a-b+b-c)^2/4=(a-c)^2/4$ $\Rightarrow$

$\left | (a-c)^3 \right |\geq 4$

WLOG:$a\geq c$

$(a-c)\geq \sqrt[3]{4}$ $\Rightarrow$

$\left | a \right | +\left | b \right |+\left | c \right |\geq \left | a \right |+\left | c \right |\geq \left | (a-c) \right |\geq \sqrt[3]{4} $

Ответ:$\sqrt[3]{4}$

пред. Правка 2   2 | проверено модератором
2016-09-14 21:51:24.0 #

Без ограничения общно­сти можем считать, что­ $a>b>c$

Пусть $a-b=x, b-c= y$­ тогда $a-c=x+y$ , то­гда $xy(x+y)=1$.

Так как ­$(x+y)^2\ge 4xy$ то $­(x+y)^3\ge 4xy(x+y)=4­$ или же $x+y\ge \sqrt[3]{4}$ дальше:

Если $b\ge 0$, то по ­треугольному неравенс­тву имеем: $|a|+|b|+|c­| =|b+x|+|b|+|y-b|\ge |b+x+b+y-b|=x+y+b\ge x+y\ge \sqrt[3]{4}$

Если $b<0$, то $|a|+|­b|+|c| =|b+x|+|-b|+|y­-b| \ge |b+x-b+y-b|=x­+y-b\ge x+y\ge \sqrt[3]{4}$.

В любом с­лучае имеем $|a|+|b|+­|c|\ge \sqrt[3]{4}$

Равенство достигается­ например при $a=\frac{1}{\sqrt[3]{2}}­ ,b=0, c= -\frac{1}{\sqrt[3]{2}}$

пред. Правка 2   3
2023-03-12 18:13:31.0 #

Пусть $a>b>c$. Заметим, что если увеличить $a,b,c$ на $x$, то условие $|(a-b)(b-c)(c-a)|=1$ сохранится. При фиксированных $a,b,c$ несложно показать, $|a+x|+|b+x|+|c+x|$ достигает минимума при $x=-b$. Далее $b=0$, откуда $$|ac(a-c)|=1\le \frac{(|a|+|c|)^3}4$$то есть $|a|+|b|+|c|\ge\sqrt[3]4$ с равенством при $a=\frac1{\sqrt[3]2},b=0,c=-\frac1{\sqrt[3]2}$.